1
$\begingroup$

Let $F$ be a number field and let $o$ be its ring of integers. Let $o_p$ resp. $F_p$ be the completion at a prime ideal $p$ in $o$. Let $B$ be the group of upper triangular matrices in $GL_2$. Let $\pi $ be a character of $B(F_p)$.

How can we describe the irreducible which occur in the restriction of the induced representations $$Res_{GL_2(o_p)} Ind_{B(F_p)}^{GL_2(F_p)} \pi = Ind_{B(o_p)}^{GL_2(o_p)} \pi = lim_r Ind_{B(o/p^r)}^{GL_2(o/p^r)} \pi.$$

Is there a nice description of the irreducibles?

$\endgroup$

1 Answer 1

4
$\begingroup$

This is done in :

Casselman, William The restriction of a representation of ${\rm GL}_2 (k)$ to ${\rm GL}_{2}({\mathfrak o})$. Math. Ann. 206 (1973), 311–318.

Also see :

Silberger, A.: $PGL_2$ over the p-adics. Lecture Notes in Mathematics 166, Berlin- Heidelberg-New York: Springer 1970

Very roughly speaking, the idea is the following. When you restrict an irreducible smooth representation of ${\rm GL}(2)$ to ${\rm GL}(2,{\mathfrak o})$ you get two types of constituents. The first ones (infinitely many) are uninteresting : they appear in the restriction of many other representations. These representations are described by Casselman. The second ones are more interesting : if an ireeducible representation contains such constituents then it must belong to a single component of the Bernstein decomposition of the category. They are called typical. Actually the situation is a bit more complicate. I've made it simpler. For more detail, you may read the paper by Henniart :

Breuil, Christophe; Mézard, Ariane Multiplicités modulaires et représentations de ${\rm GL}_2({\bf Z}_p)$ et de ${\rm Gal}(\overline{\bf Q}_p/{\bf Q}_p)$ en $l=p$. (French) [Modular multiplicities and representations of ${\rm GL}_2({\bf Z}_p)$ and ${\rm Gal}(\overline{\bf Q}_p/{\bf Q}_p)$ at $l=p$] With an appendix by Guy Henniart. Duke Math. J. 115 (2002), no. 2, 205–310.

$\endgroup$
1
  • $\begingroup$ The article of Casselman exactly provides what I need. Thanks for this nice reference. $\endgroup$
    – Marc Palm
    Apr 2, 2011 at 11:28

Your Answer

By clicking “Post Your Answer”, you agree to our terms of service and acknowledge you have read our privacy policy.

Not the answer you're looking for? Browse other questions tagged or ask your own question.